rsmorale
Thanks Received: 3
Jackie Chiles
Jackie Chiles
 
Posts: 37
Joined: February 20th, 2011
 
 
trophy
Most Thankful
 

Q7 - A neighborhood group plans to

by rsmorale Mon Aug 08, 2011 12:39 pm

I understand why the answer is A, but I want to understand a few things about answer choice B.

The question stem asks us to choose the answer that does not weaken the resident's argument.

The resident mentions two things--(i) our neighborhood already has the most residents per center of any in the city, and (ii) closing the rec center would be unacceptable, as it would preclude access to rec facilities

Answer Choice B says that children are the main users of rec facilities, and they make up a small portion of the neighborhood's population

This was a tempting answer for me. How exactly does B weaken the argument? How should I have interpreted "main" users and "disproportionately small"?
User avatar
 
demetri.blaisdell
Thanks Received: 161
LSAT Geek
 
Posts: 198
Joined: January 26th, 2011
 
 
 

Re: Q7 - A neighborhood group plans to

by demetri.blaisdell Tue Aug 09, 2011 1:06 pm

I think you've got a good handle of this question, but here at Manhattan LSAT we love those wrong answer choices. I'll run through them for you.

(B) weakens. If children are the main users of recreational facilities (in general!) and this neighborhood doesn't have very many, this gives us a reason to question the relevancy of the premise that there are more residents in this neighborhood than elsewhere. If there are only 10 children in the neighborhood and only children use these kinds of facilities, it's a great idea to close the center.

(C) weakens by saying that the center is underused. You can't argue that access to the facilities is vitally important if they are not being used often.

(D) is similar to (C). If people aren't showing up, why not close it?

(E) is a global statement about declining interest in rec centers. This will also weaken.

I hope that clears up your confusion about (B). Let me know if you have any questions.

Demetri
 
nflamel69
Thanks Received: 16
Atticus Finch
Atticus Finch
 
Posts: 162
Joined: February 07th, 2011
 
 
 

Re: Q7 - A neighborhood group plans to

by nflamel69 Fri Jul 13, 2012 4:35 pm

I don't see how D actually weakens the argument. What if the programs routinely filled at other centers are closed in this neighborhood. What if the people in this neighborhood have a different taste in activities than other places, it could be that unpopular activities at other places are really popular at this place
User avatar
 
demetri.blaisdell
Thanks Received: 161
LSAT Geek
 
Posts: 198
Joined: January 26th, 2011
 
 
 

Re: Q7 - A neighborhood group plans to

by demetri.blaisdell Fri Jul 20, 2012 5:31 pm

It's good that you are going the extra mile to think about these wrong answers, nflamel69.

I didn't fully understand your first point but it seems like you are giving me some ways that (D) might not mean that people in the neighborhood don't want to use the center. I think the word "routinely" in (D) helps address this. If they were pointing to only one class that was popular elsewhere and kept being cancelled at the center, I might agree with you. But they are talking about classes (plural) that are routinely filled. It seems odd that this center wouldn't have any interest in these classes.

If you were able to tell me that this neighborhood is mostly elderly or some special religious sect or something, that would prevent (D) from weakening. But we can't assume any of those things are true.

Maybe the way to think about it is that (D) weakens... unless we find out something more. That's enough for me to say that it weakens. But please let me know if you think I'm missing something with (D). I love to talk about wrong answer choices.

Demetri
 
patrice.antoine
Thanks Received: 35
Atticus Finch
Atticus Finch
 
Posts: 111
Joined: November 02nd, 2010
 
 
 

Re: Q7 - A neighborhood group plans to

by patrice.antoine Fri Mar 01, 2013 3:12 pm

Is (A) our correct answer because it does nothing to our argument? Is the vagueness of "a large number" the reason why this is our correct answer choice??
User avatar
 
maryadkins
Thanks Received: 641
Atticus Finch
Atticus Finch
 
Posts: 1261
Joined: March 23rd, 2011
 
 
 

Re: Q7 - A neighborhood group plans to

by maryadkins Tue Mar 05, 2013 3:21 pm

If anything (A) strengthens the resident's argument. Many of the residents can't get to a center outside of the area--so they need this one to remain!